Difference between revisions of "2005 AMC 12A Problems/Problem 17"

(Image needed)
 
Line 6: Line 6:
 
</math>
 
</math>
  
{{image}}
+
[[Image:2005 AMC 12A Problem 17.png]]
  
 
== Solution ==
 
== Solution ==
It is a [[pyramid]], so <math>\frac{1}{3} \cdot \left(\frac{1}{4}\right) \cdot (1) = \frac {1}{12}</math>.
+
It is a [[pyramid]], so <math>\frac{1}{3} \cdot \left(\frac{1}{4}\right) \cdot (1) = \frac {1}{12} \Rightarrow \boxed{(\mathrm {A})}</math>.
  
 
== See also ==
 
== See also ==

Revision as of 11:19, 15 April 2008

Problem

A unit cube is cut twice to form three triangular prisms, two of which are congruent, as shown in Figure 1. The cube is then cut in the same manner along the dashed lines shown in Figure 2. This creates nine pieces. What is the volume of the piece that contains vertex $W$?

$(\mathrm {A}) \ \frac{1}{12} \qquad (\mathrm {B}) \ \frac{1}{9} \qquad (\mathrm {C})\ \frac{1}{8} \qquad (\mathrm {D}) \ \frac{1}{6} \qquad (\mathrm {E})\ \frac{1}{4}$

2005 AMC 12A Problem 17.png

Solution

It is a pyramid, so $\frac{1}{3} \cdot \left(\frac{1}{4}\right) \cdot (1) = \frac {1}{12} \Rightarrow \boxed{(\mathrm {A})}$.

See also

2005 AMC 12A (ProblemsAnswer KeyResources)
Preceded by
Problem 16
Followed by
Problem 18
1 2 3 4 5 6 7 8 9 10 11 12 13 14 15 16 17 18 19 20 21 22 23 24 25
All AMC 12 Problems and Solutions